Prove the norm of operator derived from orthogonal projections is less or equal to 1












2












$begingroup$


Suppose $H$ is a Hilbert space. We have two orthogonal projections $P_1: Hrightarrow A_1$ and $P_2: Hrightarrow A_2$. Here $A_1$ and $A_2$ are two closed subspace of $H$. Prove that
$$
lVert P_1 - P_2rVert le 1
$$



Any hints may help. Thank you.










share|cite|improve this question









$endgroup$

















    2












    $begingroup$


    Suppose $H$ is a Hilbert space. We have two orthogonal projections $P_1: Hrightarrow A_1$ and $P_2: Hrightarrow A_2$. Here $A_1$ and $A_2$ are two closed subspace of $H$. Prove that
    $$
    lVert P_1 - P_2rVert le 1
    $$



    Any hints may help. Thank you.










    share|cite|improve this question









    $endgroup$















      2












      2








      2





      $begingroup$


      Suppose $H$ is a Hilbert space. We have two orthogonal projections $P_1: Hrightarrow A_1$ and $P_2: Hrightarrow A_2$. Here $A_1$ and $A_2$ are two closed subspace of $H$. Prove that
      $$
      lVert P_1 - P_2rVert le 1
      $$



      Any hints may help. Thank you.










      share|cite|improve this question









      $endgroup$




      Suppose $H$ is a Hilbert space. We have two orthogonal projections $P_1: Hrightarrow A_1$ and $P_2: Hrightarrow A_2$. Here $A_1$ and $A_2$ are two closed subspace of $H$. Prove that
      $$
      lVert P_1 - P_2rVert le 1
      $$



      Any hints may help. Thank you.







      linear-algebra functional-analysis






      share|cite|improve this question













      share|cite|improve this question











      share|cite|improve this question




      share|cite|improve this question










      asked Dec 3 '18 at 21:55









      SyuizenSyuizen

      934411




      934411






















          3 Answers
          3






          active

          oldest

          votes


















          1












          $begingroup$

          HINT:



          Note that it suffices to show that for any orthogonal projection $P$ on $H$, then $|P-frac{1}{2}I|=frac{1}{2}$ or $|2P-I|=1$, where $I$ is the identity operator on $H$. To show this, note that $2P-I$ is a unitary.






          share|cite|improve this answer









          $endgroup$





















            0












            $begingroup$

            You can write $A_1 = B_1oplus (A_1cap A_2)$ and $A_2=B_2oplus(A_1cap A_2)$ where $oplus$ is an orthogonal decomposition. Then
            $$
            P_{A_1}=P_{B_1}+P_{A_1cap A_2}\
            P_{A_2}=P_{B_2}+P_{A_1cap A_2},
            $$



            where the $P_{X}$ is the orthogonal projection onto $X$. So
            $$
            P_{A_1}-P_{A_2}=P_{B_1}-P_{B_2}.
            $$



            Because $B_1perp B_2$, it is easy to see that $|P_{B_1}-P_{B_2}|le 1$ because
            $$
            |P_{B_1}x-P_{B_2}x|^2=|P_{B_1}x|^2+|P_{B_1}x|^2=|P_{B_1oplus B_2}x|^2le |x|^2.
            $$






            share|cite|improve this answer









            $endgroup$









            • 1




              $begingroup$
              Why does $B_1perp B_2$ hold?
              $endgroup$
              – Syuizen
              Dec 4 '18 at 14:27



















            0












            $begingroup$

            For any $xin H$ with $|x|=1$, we have for an orthogonal projection $P$ $$langle Px,xrangle=langle P^*Px,xrangle=langle Px,Pxrangle=|Px|^2leq |Px|^2+|(I-P)x|^2=|x|^2=1.$$
            So
            $$
            langle (P_1-P_2)x,xrangle=langle P_1x,xrangle-langle P_2x,xranglein[-1,1],
            $$

            since is it a difference of two numbers each in $[0,1]$.



            For any selfadjoint operator $T$, we have $|T|=sup{|langle Tx,xrangle: |x|=1}$. So
            $$
            |P_1-P_2|=sup{|langle (P_1-P_2)x,xrangle|: |x|=1}leq1.
            $$






            share|cite|improve this answer









            $endgroup$













              Your Answer





              StackExchange.ifUsing("editor", function () {
              return StackExchange.using("mathjaxEditing", function () {
              StackExchange.MarkdownEditor.creationCallbacks.add(function (editor, postfix) {
              StackExchange.mathjaxEditing.prepareWmdForMathJax(editor, postfix, [["$", "$"], ["\\(","\\)"]]);
              });
              });
              }, "mathjax-editing");

              StackExchange.ready(function() {
              var channelOptions = {
              tags: "".split(" "),
              id: "69"
              };
              initTagRenderer("".split(" "), "".split(" "), channelOptions);

              StackExchange.using("externalEditor", function() {
              // Have to fire editor after snippets, if snippets enabled
              if (StackExchange.settings.snippets.snippetsEnabled) {
              StackExchange.using("snippets", function() {
              createEditor();
              });
              }
              else {
              createEditor();
              }
              });

              function createEditor() {
              StackExchange.prepareEditor({
              heartbeatType: 'answer',
              autoActivateHeartbeat: false,
              convertImagesToLinks: true,
              noModals: true,
              showLowRepImageUploadWarning: true,
              reputationToPostImages: 10,
              bindNavPrevention: true,
              postfix: "",
              imageUploader: {
              brandingHtml: "Powered by u003ca class="icon-imgur-white" href="https://imgur.com/"u003eu003c/au003e",
              contentPolicyHtml: "User contributions licensed under u003ca href="https://creativecommons.org/licenses/by-sa/3.0/"u003ecc by-sa 3.0 with attribution requiredu003c/au003e u003ca href="https://stackoverflow.com/legal/content-policy"u003e(content policy)u003c/au003e",
              allowUrls: true
              },
              noCode: true, onDemand: true,
              discardSelector: ".discard-answer"
              ,immediatelyShowMarkdownHelp:true
              });


              }
              });














              draft saved

              draft discarded


















              StackExchange.ready(
              function () {
              StackExchange.openid.initPostLogin('.new-post-login', 'https%3a%2f%2fmath.stackexchange.com%2fquestions%2f3024739%2fprove-the-norm-of-operator-derived-from-orthogonal-projections-is-less-or-equal%23new-answer', 'question_page');
              }
              );

              Post as a guest















              Required, but never shown

























              3 Answers
              3






              active

              oldest

              votes








              3 Answers
              3






              active

              oldest

              votes









              active

              oldest

              votes






              active

              oldest

              votes









              1












              $begingroup$

              HINT:



              Note that it suffices to show that for any orthogonal projection $P$ on $H$, then $|P-frac{1}{2}I|=frac{1}{2}$ or $|2P-I|=1$, where $I$ is the identity operator on $H$. To show this, note that $2P-I$ is a unitary.






              share|cite|improve this answer









              $endgroup$


















                1












                $begingroup$

                HINT:



                Note that it suffices to show that for any orthogonal projection $P$ on $H$, then $|P-frac{1}{2}I|=frac{1}{2}$ or $|2P-I|=1$, where $I$ is the identity operator on $H$. To show this, note that $2P-I$ is a unitary.






                share|cite|improve this answer









                $endgroup$
















                  1












                  1








                  1





                  $begingroup$

                  HINT:



                  Note that it suffices to show that for any orthogonal projection $P$ on $H$, then $|P-frac{1}{2}I|=frac{1}{2}$ or $|2P-I|=1$, where $I$ is the identity operator on $H$. To show this, note that $2P-I$ is a unitary.






                  share|cite|improve this answer









                  $endgroup$



                  HINT:



                  Note that it suffices to show that for any orthogonal projection $P$ on $H$, then $|P-frac{1}{2}I|=frac{1}{2}$ or $|2P-I|=1$, where $I$ is the identity operator on $H$. To show this, note that $2P-I$ is a unitary.







                  share|cite|improve this answer












                  share|cite|improve this answer



                  share|cite|improve this answer










                  answered Dec 3 '18 at 23:02









                  AweyganAweygan

                  14.3k21441




                  14.3k21441























                      0












                      $begingroup$

                      You can write $A_1 = B_1oplus (A_1cap A_2)$ and $A_2=B_2oplus(A_1cap A_2)$ where $oplus$ is an orthogonal decomposition. Then
                      $$
                      P_{A_1}=P_{B_1}+P_{A_1cap A_2}\
                      P_{A_2}=P_{B_2}+P_{A_1cap A_2},
                      $$



                      where the $P_{X}$ is the orthogonal projection onto $X$. So
                      $$
                      P_{A_1}-P_{A_2}=P_{B_1}-P_{B_2}.
                      $$



                      Because $B_1perp B_2$, it is easy to see that $|P_{B_1}-P_{B_2}|le 1$ because
                      $$
                      |P_{B_1}x-P_{B_2}x|^2=|P_{B_1}x|^2+|P_{B_1}x|^2=|P_{B_1oplus B_2}x|^2le |x|^2.
                      $$






                      share|cite|improve this answer









                      $endgroup$









                      • 1




                        $begingroup$
                        Why does $B_1perp B_2$ hold?
                        $endgroup$
                        – Syuizen
                        Dec 4 '18 at 14:27
















                      0












                      $begingroup$

                      You can write $A_1 = B_1oplus (A_1cap A_2)$ and $A_2=B_2oplus(A_1cap A_2)$ where $oplus$ is an orthogonal decomposition. Then
                      $$
                      P_{A_1}=P_{B_1}+P_{A_1cap A_2}\
                      P_{A_2}=P_{B_2}+P_{A_1cap A_2},
                      $$



                      where the $P_{X}$ is the orthogonal projection onto $X$. So
                      $$
                      P_{A_1}-P_{A_2}=P_{B_1}-P_{B_2}.
                      $$



                      Because $B_1perp B_2$, it is easy to see that $|P_{B_1}-P_{B_2}|le 1$ because
                      $$
                      |P_{B_1}x-P_{B_2}x|^2=|P_{B_1}x|^2+|P_{B_1}x|^2=|P_{B_1oplus B_2}x|^2le |x|^2.
                      $$






                      share|cite|improve this answer









                      $endgroup$









                      • 1




                        $begingroup$
                        Why does $B_1perp B_2$ hold?
                        $endgroup$
                        – Syuizen
                        Dec 4 '18 at 14:27














                      0












                      0








                      0





                      $begingroup$

                      You can write $A_1 = B_1oplus (A_1cap A_2)$ and $A_2=B_2oplus(A_1cap A_2)$ where $oplus$ is an orthogonal decomposition. Then
                      $$
                      P_{A_1}=P_{B_1}+P_{A_1cap A_2}\
                      P_{A_2}=P_{B_2}+P_{A_1cap A_2},
                      $$



                      where the $P_{X}$ is the orthogonal projection onto $X$. So
                      $$
                      P_{A_1}-P_{A_2}=P_{B_1}-P_{B_2}.
                      $$



                      Because $B_1perp B_2$, it is easy to see that $|P_{B_1}-P_{B_2}|le 1$ because
                      $$
                      |P_{B_1}x-P_{B_2}x|^2=|P_{B_1}x|^2+|P_{B_1}x|^2=|P_{B_1oplus B_2}x|^2le |x|^2.
                      $$






                      share|cite|improve this answer









                      $endgroup$



                      You can write $A_1 = B_1oplus (A_1cap A_2)$ and $A_2=B_2oplus(A_1cap A_2)$ where $oplus$ is an orthogonal decomposition. Then
                      $$
                      P_{A_1}=P_{B_1}+P_{A_1cap A_2}\
                      P_{A_2}=P_{B_2}+P_{A_1cap A_2},
                      $$



                      where the $P_{X}$ is the orthogonal projection onto $X$. So
                      $$
                      P_{A_1}-P_{A_2}=P_{B_1}-P_{B_2}.
                      $$



                      Because $B_1perp B_2$, it is easy to see that $|P_{B_1}-P_{B_2}|le 1$ because
                      $$
                      |P_{B_1}x-P_{B_2}x|^2=|P_{B_1}x|^2+|P_{B_1}x|^2=|P_{B_1oplus B_2}x|^2le |x|^2.
                      $$







                      share|cite|improve this answer












                      share|cite|improve this answer



                      share|cite|improve this answer










                      answered Dec 3 '18 at 23:03









                      DisintegratingByPartsDisintegratingByParts

                      59.5k42580




                      59.5k42580








                      • 1




                        $begingroup$
                        Why does $B_1perp B_2$ hold?
                        $endgroup$
                        – Syuizen
                        Dec 4 '18 at 14:27














                      • 1




                        $begingroup$
                        Why does $B_1perp B_2$ hold?
                        $endgroup$
                        – Syuizen
                        Dec 4 '18 at 14:27








                      1




                      1




                      $begingroup$
                      Why does $B_1perp B_2$ hold?
                      $endgroup$
                      – Syuizen
                      Dec 4 '18 at 14:27




                      $begingroup$
                      Why does $B_1perp B_2$ hold?
                      $endgroup$
                      – Syuizen
                      Dec 4 '18 at 14:27











                      0












                      $begingroup$

                      For any $xin H$ with $|x|=1$, we have for an orthogonal projection $P$ $$langle Px,xrangle=langle P^*Px,xrangle=langle Px,Pxrangle=|Px|^2leq |Px|^2+|(I-P)x|^2=|x|^2=1.$$
                      So
                      $$
                      langle (P_1-P_2)x,xrangle=langle P_1x,xrangle-langle P_2x,xranglein[-1,1],
                      $$

                      since is it a difference of two numbers each in $[0,1]$.



                      For any selfadjoint operator $T$, we have $|T|=sup{|langle Tx,xrangle: |x|=1}$. So
                      $$
                      |P_1-P_2|=sup{|langle (P_1-P_2)x,xrangle|: |x|=1}leq1.
                      $$






                      share|cite|improve this answer









                      $endgroup$


















                        0












                        $begingroup$

                        For any $xin H$ with $|x|=1$, we have for an orthogonal projection $P$ $$langle Px,xrangle=langle P^*Px,xrangle=langle Px,Pxrangle=|Px|^2leq |Px|^2+|(I-P)x|^2=|x|^2=1.$$
                        So
                        $$
                        langle (P_1-P_2)x,xrangle=langle P_1x,xrangle-langle P_2x,xranglein[-1,1],
                        $$

                        since is it a difference of two numbers each in $[0,1]$.



                        For any selfadjoint operator $T$, we have $|T|=sup{|langle Tx,xrangle: |x|=1}$. So
                        $$
                        |P_1-P_2|=sup{|langle (P_1-P_2)x,xrangle|: |x|=1}leq1.
                        $$






                        share|cite|improve this answer









                        $endgroup$
















                          0












                          0








                          0





                          $begingroup$

                          For any $xin H$ with $|x|=1$, we have for an orthogonal projection $P$ $$langle Px,xrangle=langle P^*Px,xrangle=langle Px,Pxrangle=|Px|^2leq |Px|^2+|(I-P)x|^2=|x|^2=1.$$
                          So
                          $$
                          langle (P_1-P_2)x,xrangle=langle P_1x,xrangle-langle P_2x,xranglein[-1,1],
                          $$

                          since is it a difference of two numbers each in $[0,1]$.



                          For any selfadjoint operator $T$, we have $|T|=sup{|langle Tx,xrangle: |x|=1}$. So
                          $$
                          |P_1-P_2|=sup{|langle (P_1-P_2)x,xrangle|: |x|=1}leq1.
                          $$






                          share|cite|improve this answer









                          $endgroup$



                          For any $xin H$ with $|x|=1$, we have for an orthogonal projection $P$ $$langle Px,xrangle=langle P^*Px,xrangle=langle Px,Pxrangle=|Px|^2leq |Px|^2+|(I-P)x|^2=|x|^2=1.$$
                          So
                          $$
                          langle (P_1-P_2)x,xrangle=langle P_1x,xrangle-langle P_2x,xranglein[-1,1],
                          $$

                          since is it a difference of two numbers each in $[0,1]$.



                          For any selfadjoint operator $T$, we have $|T|=sup{|langle Tx,xrangle: |x|=1}$. So
                          $$
                          |P_1-P_2|=sup{|langle (P_1-P_2)x,xrangle|: |x|=1}leq1.
                          $$







                          share|cite|improve this answer












                          share|cite|improve this answer



                          share|cite|improve this answer










                          answered Dec 4 '18 at 1:59









                          Martin ArgeramiMartin Argerami

                          127k1182183




                          127k1182183






























                              draft saved

                              draft discarded




















































                              Thanks for contributing an answer to Mathematics Stack Exchange!


                              • Please be sure to answer the question. Provide details and share your research!

                              But avoid



                              • Asking for help, clarification, or responding to other answers.

                              • Making statements based on opinion; back them up with references or personal experience.


                              Use MathJax to format equations. MathJax reference.


                              To learn more, see our tips on writing great answers.




                              draft saved


                              draft discarded














                              StackExchange.ready(
                              function () {
                              StackExchange.openid.initPostLogin('.new-post-login', 'https%3a%2f%2fmath.stackexchange.com%2fquestions%2f3024739%2fprove-the-norm-of-operator-derived-from-orthogonal-projections-is-less-or-equal%23new-answer', 'question_page');
                              }
                              );

                              Post as a guest















                              Required, but never shown





















































                              Required, but never shown














                              Required, but never shown












                              Required, but never shown







                              Required, but never shown

































                              Required, but never shown














                              Required, but never shown












                              Required, but never shown







                              Required, but never shown







                              Popular posts from this blog

                              Biblatex bibliography style without URLs when DOI exists (in Overleaf with Zotero bibliography)

                              ComboBox Display Member on multiple fields

                              Is it possible to collect Nectar points via Trainline?